LSAT and Law School Admissions Forum

Get expert LSAT preparation and law school admissions advice from PowerScore Test Preparation.

 Administrator
PowerScore Staff
  • PowerScore Staff
  • Posts: 8916
  • Joined: Feb 02, 2011
|
#39217
Complete Question Explanation
(The complete setup for this game can be found here: lsat/viewtopic.php?t=12911)

The correct answer choice is (C)

This Global, Must Be True question directly tests the main inference in this game, which results from the contrapositive of the second rule: if P were not used, then O could not be used, which leaves only four thread colors available for the three rugs. However, since exactly five colors must be used, we need to ensure that P is always used in one of the rugs. This prephrase is key to quickly identifying the correct answer choice (C).

Answer choice (A): This answer choice is incorrect, because F could be used in a multicolored rug without violating any of the rules.

Answer choice (B): This answer choice is incorrect, because T could be used in a multicolored rug without violating any of the rules, as shown in the 2-2-1 distribution.

Answer choice (C): This is the correct answer choice.

Answer choice (D): This answer choice is incorrect, because T need not be used in any of the rugs as long as we adhere to the 3-1-1 distribution:
PT74 - Game_#3_#12_diagram 1.png
Answer choice (E): This answer choice is also incorrect, because Y need not be used in any of the rugs as long as we adhere to the 3-1-1 distribution:
PT74 - Game_#3_#12_diagram 2.png
You do not have the required permissions to view the files attached to this post.

Get the most out of your LSAT Prep Plus subscription.

Analyze and track your performance with our Testing and Analytics Package.